Last visit was: 25 Apr 2024, 23:12 It is currently 25 Apr 2024, 23:12

Close
GMAT Club Daily Prep
Thank you for using the timer - this advanced tool can estimate your performance and suggest more practice questions. We have subscribed you to Daily Prep Questions via email.

Customized
for You

we will pick new questions that match your level based on your Timer History

Track
Your Progress

every week, we’ll send you an estimated GMAT score based on your performance

Practice
Pays

we will pick new questions that match your level based on your Timer History
Not interested in getting valuable practice questions and articles delivered to your email? No problem, unsubscribe here.
Close
Request Expert Reply
Confirm Cancel
SORT BY:
Date
Tags:
Show Tags
Hide Tags
Senior Manager
Senior Manager
Joined: 07 Dec 2017
Posts: 325
Own Kudos [?]: 1664 [13]
Given Kudos: 348
GMAT 1: 650 Q50 V28
GMAT 2: 720 Q49 V40
Send PM
Senior Manager
Senior Manager
Joined: 07 Dec 2017
Posts: 325
Own Kudos [?]: 1664 [1]
Given Kudos: 348
GMAT 1: 650 Q50 V28
GMAT 2: 720 Q49 V40
Send PM
Manager
Manager
Joined: 01 Feb 2018
Posts: 57
Own Kudos [?]: 18 [0]
Given Kudos: 157
Location: India
Concentration: Entrepreneurship, Marketing
GPA: 4
WE:Consulting (Consulting)
Send PM
Manager
Manager
Joined: 01 Feb 2018
Posts: 57
Own Kudos [?]: 18 [0]
Given Kudos: 157
Location: India
Concentration: Entrepreneurship, Marketing
GPA: 4
WE:Consulting (Consulting)
Send PM
Re: Dr Khan: Professor Bums recognizes that recent [#permalink]
AshutoshB wrote:
Must Be True. The correct answer choice is (C).

The stimulus presents a series of observations made by Dr. Khan regarding Professor Burns. An overt argument is never stated. Since the distinction between fact and opinion is crucial on the GMAT, especially in Must Be True stimuli where the correct answer choice must satisfy the strict parameters of the Prove Test, it is important to analyze each claim as follows:

Earlier observations showed a comet reservoir far out in the solar system, but recent observations cannot confirm this. (Fact)
Professor Burns interprets the recent observations as proof that the comet reservoir doesn’t exist. (Opinion)
The recent observations occurred under poor conditions. (Fact)

Note the use of double negatives in the second sentence of the stimulus: According to Prof. Burns, “this non-confirmation is enough to show that the earlier observations are incorrect.” It is imperative to immediately simplify this statement by removing both negations and distill its meaning: Burns doesn’t believe there is a comet reservoir far out in our solar system. However, since the recent observations occurred under poor conditions, it would be reasonable to doubt their significance: while they cannot prove (or disprove) the existence of a comet reservoir either way, they certainly cast doubt on Professor Burns’s conclusion. In other words, such a reservoir could exist. This prephrase agrees with answer choice (C).

Answer choice (A): Whether the recent observations, if made under good conditions, would have provided conclusive evidence of a comet reservoir is impossible to know. Beware of hypotheticals and speculative claims when answering Must Be True questions: more often than not, such claims are impossible to prove.

Answer choice (B): Just because Professor Burns reached an invalid conclusion does not mean that the recent observations actually confirm the earlier ones, i.e. that a comet reservoir exists. As stated earlier, it is impossible to know from the information provided whether or not such a reservoir actually exists. This answer choice conflates fact and opinion, and is therefore incorrect.

Answer choice (C): This is the correct answer choice. All we know from the facts presented in the stimulus is that the recent observations do not provide a suitable basis for rejecting the earlier ones. Professor Burns, who believes otherwise, is clearly wrong in making her assessment. In other words, her claims about the implications of the recent observations is incorrect—a comet reservoir could exist (we just don’t know).

Answer choice (D): As with answer choice (A), we have a hypothetical, speculative claim that cannot be proven with absolute certainty. It is entirely possible, however unlikely, that if the recent observations had been made under good conditions, they would have been enough to disprove the earlier ones.

Answer choice (E): This answer choice contains an exaggeration (“worthless”) that cannot be substantiated with the information provided. Eliminate.



is there a specific strategy to deal with such questions?? if yes pls share.

though i could crack this one.. i err while answer such questions.


Cheers!!
700 = Nirvana (short term)
Director
Director
Joined: 20 Sep 2016
Posts: 559
Own Kudos [?]: 933 [0]
Given Kudos: 632
Location: India
Concentration: Strategy, Operations
GPA: 3.6
WE:Operations (Consumer Products)
Send PM
Re: Dr Khan: Professor Bums recognizes that recent [#permalink]
chetan2u mikemcgarry GMATNinja DmitryFarber VeritasPrepBrian GMATNinjaTwo VeritasKarishma

the most probable inference we can draw is that prof. Bumms MAY BE incorrect in her conclusions. She may very well be correct the poor conditions of the exp do not render the observations worthless.. she may very well be correct..
But choice C says that she is definitely WRONG...please respond
Intern
Intern
Joined: 24 Feb 2018
Posts: 26
Own Kudos [?]: 25 [0]
Given Kudos: 43
Location: India
GPA: 3.35
WE:Military Officer (Military & Defense)
Send PM
Re: Dr Khan: Professor Bums recognizes that recent [#permalink]
Dr Khan: Professor Bums recognizes that recent observations fail to confirm earlier ones that apparently showed a comet reservoir far out in our solar system. She claims this non-confirmation is enough to show that the earlier observations are incorrect. But the recent observations occurred under poor conditions.

Which one of the following is most supported by Dr. Khan's statements?


A. If the recent observations had been made under good conditions, they would have provided conclusive evidence of a comet reservoir far out in our solar system.

B. Contrary to Professor Bums's view, the recent observations confirm the earlier ones.

C. Professor Bums's claim about the implications of the recent observations is incorrect.

D. The recent observations, even if they had been made under good conditions, would not have been enough to suggest that the earlier ones are incorrect.

E.CThe poor conditions present during recent observations render them worthless.


+1 for C
GMAT Club Verbal Expert
Joined: 13 Aug 2009
Status: GMAT/GRE/LSAT tutors
Posts: 6921
Own Kudos [?]: 63670 [1]
Given Kudos: 1774
Location: United States (CO)
GMAT 1: 780 Q51 V46
GMAT 2: 800 Q51 V51
GRE 1: Q170 V170

GRE 2: Q170 V170
Send PM
Re: Dr Khan: Professor Bums recognizes that recent [#permalink]
1
Kudos
Expert Reply
AdityaHongunti wrote:
the most probable inference we can draw is that prof. Bumms MAY BE incorrect in her conclusions. She may very well be correct the poor conditions of the exp do not render the observations worthless.. she may very well be correct..
But choice C says that she is definitely WRONG...please respond

Quote:
Dr Khan:

    Professor Bums recognizes that recent observations fail to confirm earlier ones that apparently showed a comet reservoir far out in our solar system. She claims this non-confirmation is enough to show that the earlier observations are incorrect. But the recent observations occurred under poor conditions.

The question asks us which of the choices is most supported by Dr. Khan's statements.

Quote:
C. Professor Bums's claim about the implications of the recent observations is incorrect.

  • Yes, based on the passage, we'd infer that Dr. Khan is calling Dr. Bums' statement into doubt (not necessarily saying that they're outright wrong).
  • Yes, (C) states that Professor Bums's claim is incorrect.
  • And yes, (C) is more supported by Dr. Khan's statements than (A), (B), (D), and (E).

When we choose (C), we are not stating that Dr. Khan calls Professor Bums's claim incorrect. We're only saying that (C) is more supported by Dr. Khan's statements than every other choice is. Dr. Khan is leaning towards calling Professor Bums's claim incorrect by calling into doubt the observations underlying Bums's claim. And in the meantime, every other answer is choice is much less supported by Dr. Khan's statements.

This is why (C) remains the best choice.
User avatar
Non-Human User
Joined: 01 Oct 2013
Posts: 17227
Own Kudos [?]: 848 [0]
Given Kudos: 0
Send PM
Re: Dr Khan: Professor Bums recognizes that recent [#permalink]
Hello from the GMAT Club VerbalBot!

Thanks to another GMAT Club member, I have just discovered this valuable topic, yet it had no discussion for over a year. I am now bumping it up - doing my job. I think you may find it valuable (esp those replies with Kudos).

Want to see all other topics I dig out? Follow me (click follow button on profile). You will receive a summary of all topics I bump in your profile area as well as via email.
GMAT Club Bot
Re: Dr Khan: Professor Bums recognizes that recent [#permalink]
Moderators:
GMAT Club Verbal Expert
6921 posts
GMAT Club Verbal Expert
238 posts
CR Forum Moderator
832 posts

Powered by phpBB © phpBB Group | Emoji artwork provided by EmojiOne